A boy cyclist and a girl cyclist are 10 miles apart and pedaling toward each other. The boy's rate is 6 miles per hour, and the girl's rate is 4 miles per hour. There is also a friendly fly zooming continuously back and forth from one bike to the other. If the fly's rate is 5 miles per hour, by the time the cyclists reach each other, how far does the fly fly?

Answers

Answer 1

Answer:

Below

Step-by-step explanation:

This is not as complicated as it seems ....

The cyclists combined speed is (6+4) = 10 mi/hr

   they will cover the 10 miles between them in   10 miles / 10 mi/hr = 1 hr

So the Fly  buzzes around at 5 mi/hr  and covers 5 miles in the one hour.


Related Questions

if we want to measure the magnitude of the orbital angular momentum and the projection of the angular momentum in

Answers

To measure the magnitude of the orbital angular momentum L and the projection of the angular momentum along a particular axis (say, the z-axis), we need to use the mathematical formalism of quantum mechanics.

In quantum mechanics, the orbital angular momentum L of a particle is an operator that acts on the wave function describing the particle's motion. Similarly, the z-component of the angular momentum Lz is also an operator.

The magnitude of the orbital angular momentum L can be calculated from the components of the angular momentum operator using the expression:

L^2 = L₁^2 + L₂^2 + L₃^2

where L₁, L₂, and L₃ are the x, y, and z components of the angular momentum operator, respectively. To measure the magnitude of the orbital angular momentum, we would need to measure the values of L₁ L₂, and L₃ and use them to calculate L^2.

The projection of the angular momentum along a particular axis (say, the z-axis) is given by the operator L₃. To measure the z-component of the angular momentum, we would need to measure the value of L₃ for a particular state of the system.

In practice, these measurements are often carried out using experiments involving the interaction of particles with magnetic fields. The behavior of the particles in the magnetic field allows us to infer information about the angular momentum of the particles.

The measurement of the magnitude and projection of angular momentum is an important part of many areas of physics, including quantum mechanics and solid-state physics.

To learn more about angular momentum click on,

https://brainly.com/question/29563080

#SPJ4

The distance traveled in the time interval 0 ≤ t ≤ 6, given the velocity function v(t) = t^2 - 6t - 16, can be determined by calculating the definite integral of the absolute value of the velocity function over the given time interval. The result is 128 units of distance.

To find the distance traveled in the given time interval, we need to integrate the absolute value of the velocity function v(t) = t^2 - 6t - 16 over the interval 0 ≤ t ≤ 6. The reason for taking the absolute value is that distance is a scalar quantity and does not depend on the direction of motion.

Taking the integral of the absolute value of the velocity function, we have:

∫|v(t)| dt = ∫|t^2 - 6t - 16| dt

To evaluate this integral, we need to split it into intervals where the velocity function is positive and negative. The absolute value function essentially removes the negative sign from the expression inside the absolute value brackets.

Next, we find the points where the velocity function changes sign by setting v(t) = 0:

t^2 - 6t - 16 = 0

Solving this quadratic equation, we find t = -2 and t = 8 as the points where the velocity changes sign.

Now, we evaluate the integral over the intervals [0, 2] and [2, 6] separately, considering the absolute value of the velocity function within each interval.

∫|v(t)| dt = ∫(t^2 - 6t - 16) dt over [0, 2] + ∫(-(t^2 - 6t - 16)) dt over [2, 6]

Evaluating the definite integrals, we obtain:

(128/3) + (128/3) = 256/3

Therefore, the distance traveled in the time interval 0 ≤ t ≤ 6 is 256/3 units of distance, which is approximately 85.33 units of distance.

Learn more about velocity function here:

https://brainly.com/question/30656024

#SPJ11

Question 5.4° A school with 20 professors forms 10 committees, each containing 6 profes- sors, such that every professor is on exactly 3 committees. Prove that it is possible to select a distinct representative from each committee.

Answers

Since there are 20 professors and each professor is on exactly 3 committees, there must be a total of 60 committee positions.

Let's assume that it is not possible to select a distinct representative from each committee. This means that there must be some committee that does not have a distinct representative.

Let's consider the professors on this committee. Since each professor is on exactly 3 committees, each of the professors on this committee must also be on 2 other committees.

Let's consider the 2 other committees for each of the professors on this committee. Since there are 6 professors on the committee, there are a total of 12 other committees to consider.

Each of these 12 committees must have a representative chosen from the remaining 14 professors, since we have assumed that it is not possible to select a distinct representative from the original 10 committees.

However, since there are only 14 professors remaining, and each professor can only be on 3 committees, it is not possible to choose a representative from all 12 of these committees without choosing at least one professor to be on 4 committees. This is a contradiction, since we have assumed that each professor is on exactly 3 committees.

Therefore, our assumption that it is not possible to select a distinct representative from each committee must be false, and it is indeed possible to do so.

You can read more about probability at https://brainly.com/question/24756209

#SPJ4

Please help !! I need help I can’t figure it out

Answers

Answer:

i cant see

Step-by-step explanation:

it, its to tiny

I need help with this question 632/572(127+276)

Answers

Hi! The answer would be 445.272727273

Glad I can help :)

An entry-level civil engineer earns an average bi-weekly net pay of $2,250.65. The engineer has created a monthly budget using the following percentages for expenses: Percent Housing 25% Food/Household 20% Savings 10% Transportation 5% Debt 15% Entertainment 5% Medical/Personal Care 5% Giving 5% Clothing 2% Miscellaneous 8% Which balance sheet correctly represents the engineer's income, expenses, and balance?

Answers

The engineer's balance sheet would show a monthly income of $9,002.60 ($4,501.30 x 2), total expenses of $4,001.36, and a balance of $499.94.

What is a balance sheet?

Balance sheet is a financial statement that contains details of a company's assets or liabilities at a specific point in time.

First we can use the given information to create the following balance sheet for the civil engineer:

INCOME:

Bi-weekly net pay: $2,250.65 x 2 = $4,501.30 (this assumes two pay periods per month)

EXPENSES:

Housing: 25% of $4,501.30 = $1,125.33Food/Household: 20% of $4,501.30 = $900.26Savings: 10% of $4,501.30 = $450.13Transportation: 5% of $4,501.30 = $225.07Debt: 15% of $4,501.30 = $675.20Entertainment: 5% of $4,501.30 = $225.07Medical/Personal Care: 5% of $4,501.30 = $225.07Giving: 5% of $4,501.30 = $225.07Clothing: 2% of $4,501.30 = $90.03Miscellaneous: 8% of $4,501.30 = $360.10

     TOTAL EXPENSES: $4,001.36

BALANCE: $4,501.30 - $4,001.36 = $499.94

Therefore, the engineer's balance sheet would show a monthly income of $9,002.60 ($4,501.30 x 2), total expenses of $4,001.36, and a balance of $499.94.

Learn more about balance sheet here : brainly.com/question/1113933

#SPJ1

Maximize Z = 7X1 + 5X2
Subject to X1 + 2X2 ≤ 6
4X1 + 3X2 ≤ 12
X1, X2 ≥ 0

Answers

The solution is [tex]x_1[/tex] = 3 and [tex]x_2[/tex]= 0.

What is Simplex Method?

Simplex method is an approach to solving linear programming models by hand using slack variables, tableaus, and pivot variables as a means to finding the optimal solution of an optimization problem.

Given:

We transfer the row with the resolving element from the previous table into the current table, elementwise dividing its values ​​into the resolving element:

[tex]P_2[/tex] = [tex]P_2[/tex] / [tex]x_{2,1[/tex] = 12 / 4 = 3;

[tex]x_{2,1[/tex] = [tex]x_{2,1[/tex] / [tex]x_{2,1[/tex] = 4 / 4 = 1;

[tex]x_{2,2[/tex] = [tex]x_{2,2[/tex] / [tex]x_{2,1[/tex] = 3 / 4 = 0.75;

[tex]x_{2,3[/tex] = [tex]x_{2,3[/tex] / [tex]x_{2,1[/tex] = 0 / 4 = 0;

[tex]x_{2,4[/tex] = [tex]x_{2,4[/tex] / [tex]x_{2,1[/tex] = 1 / 4 = 0.25;

The remaining empty cells, except for the row of estimates and the column Q, are calculated using the rectangle method, relative to the resolving element:

[tex]P_1[/tex] = ([tex]P_1[/tex] . [tex]x_{2,1[/tex]) - ([tex]x_{1,1[/tex] . [tex]P_2[/tex]) / [tex]x_{2,1[/tex]= ((6 x 4) - (1 x 12)) / 4 = 3

[tex]x_{1,1[/tex]= (([tex]x_{1,1[/tex]. [tex]x_{2,1[/tex]) - ([tex]x_{1,1[/tex] . [tex]x_{2,1[/tex])) / [tex]x_{2,1[/tex]= ((1 x 4) - (1 x 4)) / 4 = 0

[tex]x_{1,3[/tex]= (([tex]x_{1, 3[/tex]. [tex]x_{2,1[/tex]) - ([tex]x_{1,1[/tex] . [tex]x_{2,3[/tex])) / [tex]x_{2,1[/tex]= ((1 x 4) - (1 x 0)) / 4 = 1

[tex]x_{1,4[/tex] = (([tex]x_{1, 4[/tex]. [tex]x_{2,1[/tex]) - ([tex]x_{1,1[/tex]. [tex]x_{2,4[/tex])) / [tex]x_{2,1[/tex]= ((0 x 4) - (1 x 1)) / 4 = -0.25

Objective function value

We calculate the value of the objective function by elementwise multiplying the column [tex]C_b[/tex] by the column P, adding the results of the products.

Max P = (C[tex]b_1[/tex] x [tex]P_{01[/tex]) + (C[tex]b_{11[/tex] x [tex]P_2[/tex])= (0 x 3) + (7 x 3) = 21;

Evaluated Control Variables

We calculate the estimates for each controlled variable, by element-wise multiplying the value from the variable column, by the value from the Cb column, summing up the results of the products, and subtracting the coefficient of the objective function from their sum, with this variable.

Max [tex]x_1[/tex]= ((C[tex]b_1[/tex] . [tex]x_{1,1[/tex]) + (C[tex]b_2[/tex] . [tex]x_{2,1[/tex]) ) - k[tex]x_1[/tex] = ((0 x 0) + (7 x 1) ) - 7 = 0

Max [tex]x_2[/tex]= ((C[tex]b_1[/tex] . [tex]x_{1,2[/tex]) + (C[tex]b_2[/tex] . [tex]x_{2,2[/tex]) ) - k[tex]x_2[/tex] = ((0 x 1.25) + (7 x 0.75) ) - 5 = 0.25

Max [tex]x_3[/tex]= ((C[tex]b_1[/tex] . [tex]x_{1,3[/tex]) + (C[tex]b_2[/tex] . [tex]x_{2,3[/tex]) ) - k[tex]x_3[/tex] = ((0 x 1) + (7 x 0) ) - 0 = 0

Max [tex]x_4[/tex]= ((C[tex]b_1[/tex] . [tex]x_{1,4[/tex]) + (C[tex]b_2[/tex] . [tex]x_{2,4[/tex]) ) - k[tex]x_4[/tex] = ((0 x -0.25) + (7 x 0.25) ) - 0 = 1.75

Since there are no negative values ​​among the estimates of the controlled variables, the current table has an optimal solution.

The value of the objective function:

F = 21

The variables that are present in the basis are equal to the corresponding cells of the column P, all other variables are equal to zero:

[tex]x_1[/tex] = 3;

[tex]x_2[/tex]= 0;

Learn more about Simplex Method here:

https://brainly.com/question/30387091

#SPJ9

I need help on confused I don’t get it once so ever

Answers

The value of x is given as follows:

x = 34.

The angle measures are given as follows:

<B = 102º. <C = 78º.

How to obtain the angle measures?

In a parallelogram, consecutive angles are supplementary, meaning that the sum of their measures is of 180º.

Angles A and B are consecutive, hence the value of x is obtained as follows:

2x + 10 + 3x = 180

5x = 170

x = 34.

Then the angle measures are given as follows:

<B = 3(34) = 102º. <C = 78º. (consecutive with B, 180 - 102 = 78).

More can be learned about angle measures at https://brainly.com/question/2046046

#SPJ1

5
6
Check
7
8
10
11
12
13
A-, B-, C- 51.1
14
15
Consider a triangle ABC like the one below. Suppose that C-96°, a-33, and b=39. (The figure is not drawn to scale.) Solve the triangle.
Carry your intermediate computations to at least four decimal places, and round your answers to the nearest tenth.
If there is more than one solution, use the button labeled "or".
DSD
X
16
No
solution
5
Save For Later
© 2023 McGraw Hill LLC. All Rights Reserved. Terms of Use |

Answers

The value of C is 108°

How to solve for this

Given [tex]a=33,\ \ b=26, \ \ \angle B=31^0 .[/tex]

We have to find [tex]\angle C, \ \angle A \ and\ c[/tex]

We can use the cosine formula to find these:

[tex]cosB=\frac{a^2+c^2-b^2}{2ac}\\cos31^o=\frac{33^2+c^2-26^2}{2(33)(c)}\\0.8572=\frac{413+c^2}{66c}\\56.5752c=413+c^2\\i.e. c^2-56.5752c+413=0\\\Rightarrow c=47.9647, \ 8.6104\\So, \mathbf{c=48 \ or \ c=9}\\Now if c=48:\\cosA=\frac{b^2+c^2-a^2}{2bc}\\=\frac{26^2+48^2-33^2}{2(26)(48)}\\[/tex]

[tex]=\frac{1891}{2496}\\=0.75761\\\therefore A=cos^{-1}(0.75761)=40.746^0\approx 41^0\\i.e. \mathbf{\angle A= 41^0}\\cosC=\frac{a^2+b^2-c^2}{2ab}\\=\frac{33^2+26^2-48^2}{2(33)(26)}\\=\frac{-539}{1716}\\=-0.3141\\\therefore C=cos^{-1}(-0.3141)=108.306^0\approx 108^0[/tex]

Read more about geometry here:

https://brainly.com/question/19241268

#SPJ1

I NEED HELP PLEASE HELP ME

Answers

Least to greatest:

6 1/4, √ 40, 13/2
2nd one is first one last is second, first last

What is the equation of the line that passes through the point (-8, 6) and has a slope
of -1/4?

Answers

Answer:

= 4y + x - 16 = 0.

Step-by-step explanation:

Point are (-8,6)

where

x1 = -8 and y1 = 6

the equation o the line is given by the formula

[tex] = \frac{y - y1}{x - x1} = m[/tex]

where (m) is the gradient

the gradient (m) = -1/4

therefore the equation of the line

[tex] \frac{y - y1}{x - x1} = m \\ = \frac{y - 6}{x - ( - 8)} = \frac{ - 1}{4} \\ = \frac{y - 6}{x + 8} = \frac{ - 1}{4} \\ = 4(y - 6) = - 1(x + 8) \\ = 4y - 24 = - x - 8 \\ = 4y = - x - 8 + 24 \\ = 4y = - x + 16 \\ = 4y + x - 16 = 0.[/tex]

therefore the equation of the line is = 4y + x - 16 = 0.

note// y and x where not given any value.

For 3y-2x=-18 determine the value of y when x = 0, and the value of x when y = 0

Answers

3y-2x=-18
3y-2(0)=-18 (sub in x with 0 to find y)
3y=-18 (anything times 0 is 0)
Divide both sides of the equation by 3 to get y alone
Y=-6

3y-2x=-18
3(0)-2x=-18 (sub in y with 0 to find y)
-2x=-18 (anything times 0 is 0)
Divide both sides of the equal sign by -2 to get x alone
X=9 (negative divided by a negative will become positive)

Hope this helps!!

A young family is looking to buy a house and decides they need at least a 3/4-acre lot so there’s room for the kids and their dog Moose to play. Their agent plans to show them a house with a 30,000-sq-ft lot. Will she be wasting her time?

Answers

Answer: We can calculate the size of the lot in acres and then compare it to the minimum size of 3/4 of an acre that the family is looking for.

1 acre = 43,560 square feet

So, a 30,000 sq-ft lot is equivalent to:

30,000 sq-ft / 43,560 sq-ft/acre = 0.6864 acres

Since 0.6864 acres is less than 3/4 of an acre, the family's agent will be wasting her time by showing them the house with a 30,000 sq-ft lot. The lot is not big enough for the family's needs.

Step-by-step explanation:

Cecily purchases a box of 100 paper clips. She puts 37 /100 of the paper clips in a jar on her desk and puts another 6 /10 in her drawer at home. Shade a grid that shows how many of the paper clips are in Cecily's jar and drawer, then write the fraction tbe grid represents.

Answers

The fraction of the total of paper clips Cecily put in the jar and drawer is [tex]\frac{97}{100}[/tex] (see the attachment below).

How many clips did Cecily put in the jar and drawer?

Paper clips in the jar: 37/100, which means Cecily put 37 clips in the jar.

Paper clips in the drawer: 6/10, now let's find out the number of clips this fraction is equivalent to:

100 (total clips) / 10 x 6 = 60

Total clips: 60 +37= 97. This number of clips can be expressed as [tex]\frac{97}{100}[/tex] .

How to represent this in a grid?

To represent this in a grid color a total of 97 squares in a grid with 100 squares as it is shown below.

Learn more about fractions in https://brainly.com/question/10354322

#SPJ1

In each of the following systems, find conditions on a, b, and c for which the system has solutions: (a)3x + 2y - z=a x + y + 2z = b 5x+4y + 32=c (b) -3x + 2y + 4z = a
- x - 2y + 3z = b
-X -6y + 232 = 0 (C)4x - 2y + 3z = a 2x - 3y – 2z = b 4x - 2y + 32=c

Answers

The conditions on a, b, and c for which the Linear equations has solutions are det(A) ≠ 0,  Rank[A|B] = Rank(A).

(a) 3x + 2y - z = a

x + y + 2z = b

5x + 4y + 32 = c

For this system to have solutions, the augmented matrix [A|B]  must have a unique solution. This is equivalent to determinant of the coefficient matrix A is non-zero, and the system is consistent (the row rank of the augmented matrix is equal to the row rank of the coefficient matrix). Therefore, the conditions on a, b, and c for this system to have solutions are:

det(A) ≠ 0

Rank[A|B] = Rank(A)

(b) -3x + 2y + 4z = a

-x - 2y + 3z = b

-x - 6y + 232 = 0

For this system to have solutions, the conditions are the same as in (a):

det(A) ≠ 0

Rank[A|B] = Rank(A)

(c) 4x - 2y + 3z = a

2x - 3y – 2z = b

4x - 2y + 32 = c

For this system to have solutions, the conditions are the same as in (a) and (b):

det(A) ≠ 0

Rank[A|B] = Rank(A)

For each system of linear equations to have solutions, the number of equations must be equal to the number of variables, and the determinant of the coefficient matrix must be non-zero.

To know more about Linear equations:

https://brainly.com/question/13041800

#SPJ4

1.28 Ф Vector operations and units. (Chapter 2) Circle the vector operations below (scalar multiplication, addition, dot-product, etc.) that are defined for a position vector a (with units of m) and a velocity vector b (with units of P. a+b 1.29 Using vector identities to simplify expressions (refer to Homework 1.13). One reason to treat vectors as basis-independent quantities is to simplify vector expressions with- out resolving the vectors into orthogonal x, y, z or i k components. Simplify the following Vector expression (3u-29) x (u + v) vector expressions using various properties of dot-products and cross-products Simplified vector expression Express your results in terms of dot-products - and cross- products x of the arbitrary vectors υ. v. w (ie.. υ. v, w are not orthogonal). 2 1.30 Vector concepts: Solving a vector equation. (Section 2.10.5) Consider the vector equation to the right and the process that follows that solves for θ (a, is a unit vector and v,, θ, R are scalars). This process is a valid way to solve for θ. True/False Explain: vr ax 0

Answers

The vector operations defined for a position vector a and a velocity vector b are vector addition and scalar multiplication.

Simplified Vector expression is (3u - 29) x u + (3u - 29) x v.

False, as the process that follows, solving for θ by dividing by the magnitude of v and taking the inverse sine, is only valid if v x a = 0.

1.28: The vector operations defined for a position vector a and a velocity vector b are vector addition and scalar multiplication.

1.29: Using the properties of the cross-product, the simplified expression is:

3u x u + 3u x v - 29 x u - 29 x v

Using the fact that the cross-product is distributive over vector addition, we can write this as:

3u x u + 3u x v - 29 x u - 29 x v = (3u - 29) x u + (3u - 29) x v

So the simplified vector expression is (3u - 29) x u + (3u - 29) x v.

1.30: False.

The given equation,

v x (a x R) = 0,

can be simplified to a scalar triple product,

(v x a) · R = 0.

Since the scalar triple product is equal to the determinant of a matrix formed by the three vectors,

this equation implies that either v x a = 0 or R = 0.

Therefore, the process that follows, solving for θ by dividing by the magnitude of v and taking the inverse sine, is only valid if v x a = 0.

The vector operations defined for a position vector a and a velocity vector b are vector addition and scalar multiplication.

Simplified Vector expression is (3u - 29) x u + (3u - 29) x v.

False as the process that follows, solving for θ by dividing by the magnitude of v and taking the inverse sine, is only valid if v x a = 0.

For similar questions on vector operations and units,

https://brainly.com/question/26700114

#SPJ4

Steven sprinted 14 1/3 laps and then took a break by jogging 4 laps. How much farther did Steven sprint than jog?

Answers

The total number of laps Steven ran is:

14 1/3 + 4 = 57/3 + 12/3 = 69/3 = 23 laps

The distance Steven sprinted is 14 1/3 laps, which is equivalent to:

43/3 laps

The distance Steven jogged is 4 laps, which is equivalent to:

12/3 laps

Therefore, the difference between the distance Steven sprinted and jogged is:

43/3 - 12/3 = 31/3

So, Steven sprinted 31/3 laps farther than he jogged.

Thus, the answer is 31/3 laps.

If ABCD is a parallelogram,
find the value of x.
7x + 2
9x-28

Answers

7x+2=9x-28

Solve


Solution

X=15

Subtract the polynomials (35 + 83) − (72 − 63). Show all work

Answers

Answer:

35 + 83) − (72 − 63) = 109

Step-by-step explanation:

To subtract the polynomials (35 + 83) − (72 − 63), we'll simplify each term in the polynomials first and then subtract the corresponding terms:

(35 + 83) = 118

(72 - 63) = 9

Now we can subtract the corresponding terms:

118 - 9 = 109

So the result of (35 + 83) − (72 − 63) is 109.

Aɳʂɯҽɾҽԃ Ⴆყ ɠσԃKEY ꦿ

Answer:

Okay first do what is in the parenthesis:

35 + 83 = 118

72 - 63 = 9

Now subtract:

118 - 9 = 109

The answer is: 109

Step-by-step explanation:

Hope it helps!

PLEASE HELP WILL GIVE 100 PTS AND BRAINLYEST

Answers

A sequence of transformations is applied to a polygon. [ The following statements represent a sequence of transformations where the resulting polygon is similar to the original polygon but have a smaller area than the original polygon is that: a dilation about the origin by a scale factor of 2 3 followed by a rotation of 90° counterclockwise about the origin and a translation 5 units left.

A total of 14 different people were randomly surveyed and asked how many hours per day they worked the week before Their answers are included below. Construct a box and whisker plot using a TI-83. TI-83 Plus, or TI-84 graphing calculator. Au plots below have the following window settings: Xmin = 7.95. Xmax = 8.25, Xsel = 0.01, Ymin = -0.5, Ymax = 3.5, Ysel = 1, Xres=1 Average Work Hours Per Day 8.08 8.08 8.04 8.12 8.05 8.05 7.97 8.00 8.10 8.22 8.09 8.00 815 D ex 70 6 t y u g LLLL wa Lose Calculate measures of center and spread using Technology Calculator Med=8.08 P1L1 Med=8.065 Med=8.14 P1:11 Med=8.09

Answers

The boxplot for the provide data is present in above figure. Median or central tendency of data is equals to 8.08. The lower and upper limits of outliers are 7.928 and 8.248 respectively.

The first task is to compute the median and the quartiles. And, in order to compute the median and the quartiles, the data needs to be put into the ascending order, as shown in the above table. Since the sample size n = 14 is even, we have that (n+1)/2 = (14+1)/2 =7.5, is not an integer value, the median is computing by taking the average of the values at the positions 7ᵗʰ and 8ᵗʰ, as shown below Median = (8.08 + 8.08)/2 = 8.08

Quartiles : The quartiles are computed using the table with the data in the asencending order. For Q₁ we have to compute the following position:

pos(Q) = (n+1)× 25/100 = 15×25/100 = 3.75

Since 3.75 is not an integer number, Q₁ is computed by interpolating between the

values located in the 3ᵗʰ and 4ᵗʰ positions, as shown in the formula above

Q₁ = 8.04+ (3.75 - 3) (8.05 - 8.04) = 8.0475

For Q3 we have to compute the following position:

pos(Q₃) = (n + 1)× 75/100

= (14+1)×75/100 = 11.25

Since 11.25 is not an integer number, Q₃ is computed by interpolating between

the values located in the 11ᵗʰ and 12ᵗʰ positions, as shown in the formula below

Q₃ = 8.12 + (11.25 - 11) × (8.15 8.12) = 8.1275

The interquartile range is therefore

IQR = Q₃ - Q₁ = 8.1275 - 8.0475 = 0.08

Now, we can compute the lower and upper limits for outliers:

Lower = Q₁ - 1.5 x IQR = 8.0475 - 1.5×0.08 =7.928

Upper = Q₃ + 1.5 x IQR = 8.1275 + 1.5× 0.08 =8.248

and then, an outcome X is an outlier if X < 7.928, or if X > 8.248.

In this case since all the outcomes X are within the values of Lower = 7.9275 and Upper = 8.2475, then there are no outliers. The above boxplot is obtained.

To learn more about Quartiles, visit :

https://brainly.com/question/28169373

#SPJ4

please help. . . . . . . .

Answers

Answer:

3. [tex]y=(x-3)(x-2)[/tex]

4. [tex]y = (x + 2)^2[/tex]

Step-by-step explanation:

In this problem, we are asked to find the equations in their factored form of the graphed parabolas.

3. We can see that the parabola's vertex is at (3, -4). We can plug the coordinates of that point into the vertex form equation:

[tex]y=(x-a)^2 + b[/tex]

where [tex](a,b)[/tex] is the vertex of the parabola.

[tex]y=(x-3)^2 -4[/tex]

Then, we can expand the right side of the equation to an unfactored form.

[tex]y=x^2-6x+9 -4[/tex]

[tex]y=x^2-6x-5[/tex]

Finally, we can factor the right side of the equation.

[tex]\boxed{y=(x-3)(x-2)}[/tex]

4. First, input the vertex's coordinates into the vertex form equation.

[tex]y=(x - (-2))^2 + 0[/tex]

Then, simplify.

(Remember that subtracting a negative is the same as adding the positive)

[tex]\boxed{y=(x+2)^2}[/tex]

Answer question on the picture attached

Answers

The average rate of change over the interval (-2,0) is -2.

What is the average rate of change?

The average rate of change describes how quickly one quantity changes in comparison to another. It indicates how much the function changed per unit during the specified interval.

Given that the interval is (-2,0). The coordinate of the point from the interval -2,0 is ( -2 , 4 ).

The average rate of change will be calculated as:-

Rate of change = ΔY / ΔX

Rate of change = ( 4 - 0 ) ( -2 - 0 )

Rate of change = -2

Therefore, the average rate of change will be -2.

To know more about the average rate of change follow

https://brainly.com/question/8728504

#SPJ1

help please see photo​

Answers

The solution of the inequality using the coordinate pair is 5.

What is Linear Inequality?

Linear inequalities are those expressions which are connected by inequality signs like >, <, ≤, ≥ and ≠ and the value of the exponent of the variable is 1.

Given linear inequality is,

y ≤ -2x + 3

We have a coordinate pair (-1, 5).

So substitute x = -1 and y = 5 in the given inequality.

5 ≤ (-2 × -1) + 3

5 ≤ 2 + 3

5 ≤ 5

Hence the missing numbers are found.

Learn more about Linear Inequalities here :

https://brainly.com/question/19526736

#SPJ1

The surface of a table to be built will be in the shape shown below. The distance from the center of the shape to the center of each side is 10.4 inches and the length of each side is 12 inches.

A hexagon labeled ABCDEF is shown will all 6 sides equal in length. ED is labeled as 12 inches. A perpendicular is drawn from the center of the hexagon to the side ED. This perpendicular is labeled as 10.4 inches.
Part A: Describe how you can decompose this shape into triangles. (2 points)

Part B: What would be the area of each triangle? Show every step of your work. (5 points)

Part C: Using your answers above, determine the area of the table's surface. Show every step of your work.

Answers

A)  We can draw a line perpendicular to side ED from the center of the hexagon, which will bisect side ED and create two additional triangles.

B) Area of each triangle = 31.16 square inches (rounded to two decimal places)

C) Total surface area = 375.87 square inches (rounded to two decimal places)

What is Surface Area ?

Any geometric shape with three dimensions can have its surface area determined. The area or region that an object's surface occupies is known as its surface area.

Now in the given question,

Part A: To decompose this shape into triangles, we can draw lines connecting the center of the hexagon to each of its vertices, creating six triangles. We can also draw a line perpendicular to side ED from the center of the hexagon, which will bisect side ED and create two additional triangles.

Part B: Each of the six triangles created by connecting the center of the hexagon to its vertices is an equilateral triangle, because all sides of the hexagon are equal in length. The area of an equilateral triangle can be calculated using the formula:

Area = (√3 / 4) x side²

where side is the length of one side of the equilateral triangle. In this case, the length of each side of the equilateral triangle is also 12 inches, so we have:

Area = (√3 / 4) x 12²

Area = (√3 / 4) x 144

Area = 36√3 square inches

Each of the two triangles created by drawing a perpendicular from the center of the hexagon to side ED is a right triangle, because one of its angles is 90 degrees. We can use the Pythagorean theorem to calculate the length of the other two sides of the right triangle. We know that one side has length 10.4 inches and the hypotenuse (which is also a side of the hexagon) has length 12 inches. Let x be the length of the other side of the right triangle. Then we have:

x² + 10.4² = 12²

x²+ 108.16 = 144

x² = 35.84

x = √35.84

x = 5.99 inches (rounded to two decimal places)

The area of each right triangle can be calculated using the formula:

Area = (1/2) x base x height

where the base is 5.99 inches (the length of the side opposite the 90 degree angle) and the height is 10.4 inches (the length of the side adjacent to the 90 degree angle). We have:

Area = (1/2) x 5.99 x 10.4

Area = 31.16 square inches (rounded to two decimal places)

Part C: To determine the area of the table's surface, we need to add up the areas of all eight triangles. There are six equilateral triangles, each with an area of 36sqrt(3) square inches, and two right triangles, each with an area of 31.16 square inches. Therefore, the total area of the table's surface is:

Total area = 6 x 36√3 + 2 x 31.16

Total area = 216√3 + 62.32

Total area = 375.87 square inches (rounded to two decimal places)

Learn more about area, visit :

https://brainly.com/question/27683633

#SPJ1

What is the value of X and Y?

Answers

The value of x and y is 31° and 101°.  Value is the measure of what something is worth, either financially or in terms of importance.

What is meant by value?The value describes the value of each digit in relation to its position in the number. The place value and face value of the digit are multiplied to arrive at the answer. Value equals Location Value minus Face Value.Value is concerned with how much something is worth, whether in monetary terms or in terms of significance. It can mean "identify how much something is worth" like a reward valued at $200 or it can indicate "hold something in high esteem" like "I value our friendship."The things that someone values are known as their values. In other terms, values are what a person or a group of people see as "important." Examples include valor, integrity, freedom, and creativity, among others.

[tex]$\angle \mathrm{PRT}+\angle \mathrm{RTP}+\angle \mathrm{TPR}=180^{\circ} \text { (angle sum property of triangle) } \\[/tex]

[tex]& \Rightarrow \mathrm{x}+\left(180^{\circ}-\angle \mathrm{RTQ}\right)+60^{\circ}=180^{\circ} \text { (linear pair) } \\[/tex]

[tex]& \Rightarrow \mathrm{x}+\left(180^{\circ}-97^{\circ}\right)+60^{\circ}=180^{\circ} \\[/tex]

We get,

[tex]& \Rightarrow \mathrm{x}=31^{\circ} \\[/tex]

[tex]& \text { Now } \angle \mathrm{PRT}+\angle \mathrm{TRQ}+\angle \mathrm{QRS}=180^{\circ} \text { (angle of straight line) } \\[/tex]

[tex]& \Rightarrow \mathrm{x}+48^{\circ}+\mathrm{y}=180^{\circ} \\[/tex]

Simplifying,

[tex]& \Rightarrow 31^{\circ}+48^{\circ}+\mathrm{y}=180^{\circ} \\[/tex]

Then we get,

[tex]& \Rightarrow \mathrm{y}=101^{\circ}[/tex]

To learn more about value, refer to:

https://brainly.com/question/24368848

#SPJ1

indicate whether the following statements regarding the variance and standard deviation are true or false. a) the population variance and sd use n in the denominator instead of n-1. false b) the sample variance and sd inherit the strengths and weaknesses of the sample mean. _______
c) if we did not square the distances in the numerator, we would end up with 0. ______
d) the sample variance and sd are robust to outliers and/or extreme points.________

Answers

a) the population variance and sd use n in the denominator instead of n-1.  - false

b) the sample variance and sd inherit the strengths and weaknesses of the sample mean. - True

c) if we did not square the distances in the numerator, we would end up with 0. -false

d) the sample variance and sd are robust to outliers and/or extreme points.-false

a) False. The population variance and standard deviation use N in the denominator, where N is the size of the population. The sample variance and standard deviation use n-1 in the denominator, where n is the size of the sample.

b) True. The sample variance and standard deviation inherit the strengths and weaknesses of the sample mean, since they are based on the deviations of the sample values from the sample mean.

c) False. If we did not square the distances in the numerator, we would end up with the mean deviation, which is a valid measure of dispersion but not the variance.

d) False. The sample variance and standard deviation are not robust to outliers and/or extreme points, as they are sensitive to the values of the data points and can be significantly influenced by the presence of outliers.

For more questions on mean deviation and sample variance

https://brainly.com/question/15362125

#SPJ4

For each situation, determine why the situation cannot be modelled after a binomial distribution. (2 pts each)a. A bowl of candy contains 23 Pluto Bars, 21 Jolly Farmers, 14 Husky's Kisses, 21 Finnish Fishes, and 18 HairHeads. Kristina decides to pull out 77 pieces of candy, without putting any back, and count only the number of Jolly Farmers that she grabs.b. A bowl of candy contains 23 Pluto Bars, 21 Jolly Farmers, 14 Husky's Kisses, 21 Finnish Fishes, and 18 HairHeads. Ian decides to pull out 70 pieces of candy, with replacement, and record each type of candy the he pulls.c. A bowl of candy contains 23 Pluto Bars, 21 Jolly Farmers, 14 Husky's Kisses, 21 Finnish Fishes, and 18 HairHeads. Marissa decides to pull out pieces of candy, with replacement, until a Husky's Kiss is pulled.

Answers

The question is geometric distribution. We are concerned with just the first success. That means there is no fixed number of trials

How to solve this

Before a situation can model binomial distribution.

1) There should be a fixed number of trial2) Each trial should have two possible outcomes (success or failure)3) The probability of success is the same for each trial.4) The trials are independent.

For question a.

The probability of success is not the same for each trial. Since we are not putting any back. So it cannot be modeled as binomial distribution

For question b.

This has many outcomes not just success and failure.

Lan needs to record Pluto bar, jolly farm, husky kisses, finish fish, and hair heads.

For question c.

The question is geometric distribution. We are concerned with just the first success. That means there is no fixed number of trials

Read more about geometric distribution here:

https://brainly.com/question/29210110

#SPJ1

I am having a very hard time figuring this out can someone help me with #5 please

Answers

it will be B . because u have to multiply the answer . against the graph

An ordinary (fair) die is a cube with the numbers 1 through 6 on the sides (represented by painted spots). Imagine that such a die is rolled twice in succession and that the face values of the two rolls are added together. This sum is recorded as the outcome of a single trial of a random experiment.
Compute the probability of each of the following events:
Event A: The sum is greater than 6. Event B: The sum is divisible by 3 or 5 (or both).
Write your answers as exact fractions.
P(a)
P(b)

Answers

The final answers are:

a) P(A) = 7/12

b) P(B) = 11/36

To compute the probabilities of Events A and B, we can use a table to list all possible outcomes and their corresponding sums:

Die 1 Die 2 Sum

1         1         2

1         2         3

1         3         4

1         4        5

1         5        6

1         6        7

2         1        3

2         2        4

2         3        5

2         4        6

2         5        7

2         6        8

3         1        4

3         2        5

3         3        6

3         4        7

3         5        8

3         6        9

4         1        5

4         2        6

4         3        7

4         4        8

4         5        9

4         6        10

5         1         6

5         2         7

5         3         8

5         4         9

5         5         10

5         6         11

6         1          7

6         2          8

6         3               9

6           4          10

6         5          11

6         6          12

a) Event A: The sum is greater than 6. From the table, we can see that there are 21 outcomes where the sum is greater than 6 (highlighted in bold). Thus, the probability of Event A is:

P(A) = 21/36 = 7/12

b) Event B: The sum is divisible by 3 or 5 (or both). From the table, we can see that the sums that are divisible by 3 or 5 (or both) are:

3, 5, 6, 9, 10, 12

There are 11 outcomes with these sums (highlighted in bold). Thus, the probability of Event B is:

P(B) = 11/36

Therefore, the final answers are:

a) P(A) = 7/12

b) P(B) = 11/36

For more such questions on Probability: brainly.com/question/30034780

#SPJ4

contracts for two construction jobs are randomly assigned to one or more of three firms, a, b, and c. the joint distribution of y1, the number of contracts awarded to firm a, and y2, the number of contracts awarded to firm b, is given by the entries in the following table. y1 y2 0 1 2 0 1 9 2 9 1 9 1 2 9 2 9 0 2 1 9 0 0 find cov(y1, y2). (round your answer to three decimal places.) cov(y1, y2)

Answers

The value of covariance cov(y1, y2) is -0.222

                      y1      

y2         0         1         2          Total

0          1/9      2/9     1/9          4/9

1          2/9      2/9      0            4/9

2          1/9        0       0            1/9

Total   4/9       4/9     1/9          1      

  marginal distribution of y2:

y2      P(y2)      y2P(y2)      y2^2P(y2)

0         4/9        0.0000        0.0000

1          4/9         0.4444        0.4444

2          1/9         0.2222        0.4444

total      1          0.66667       0.88889

E(y2) = 0.6667

E(y2^2) = 0.8889

Var(y2) = E(y2^2)-(E(y2))^2=0.4444

marginal distribution of y1  

y1       P(y1)       y1P(y1)      y1^2P(y1)

0         4/9       0.0000      0.0000

1          4/9        0.4444       0.4444

2          1/9         0.2222      0.4444

total  1.00          0.6667      0.8889

E(y1) = 0.6667

E(y1^2) = 0.8889

Var(y1)= σy = E(y1^2)-(E(y1))^2 = 0.4444

E(XY) =∑xyP(x,y)=0.2222

Cov(y1,y2) = -0.222

To learn more about Covariance here:

brainly.com/question/14300312

#SPJ4

Other Questions
Two missing emperor tamarin monkeys found Tuesday in an abandoned house a few miles away from the zoo? 15) If a square prism has a volume of 104 cubic centimeters and a height of eight centimeters, explain how tofind the area of the side that is a square. Sally was removing her nail polish to get ready for a dance. She was also drinking a bottle of water. The doorbell rang and she rushed to answer it, leaving the open bottle of nail polish remover (acetone) and the bottle of water open. Sally forgot about them until the next morning and when she checked, the nail polish remover (acetone) was empty and the water looked untouched.Sally believes the particles in water stick together more. In terms of your knowledge of the strength of electrical forces and the structure of substances, explain Sally's statement. to americans the rise to modernity meant following the ideas of the enlightenment, which emphasized the free individual. on the one hand, the individual must be free from the tyranny of a king. on the other hand, ____ . which of the following is the appropriate sentence for the blank?the individual should be a part of the natural world. the individual should be free from the bonds of the natural world. the individual should follow the rules of nature. the individual should bond with the natural world. This person served three years in reform school for attempted burglary before writing many famous rock-and-roll songs. A.Ralph BuncheB.Duke EllingtonC.Barbara JordanD.Chuck Berry jordan peterson had an interview with vpro documentary whereupon the journalist asked a variety of questions to trap jordan peterson. True or False What is the difference between signs and symptoms examples? when a cloud forms, latent heat is ____ the atmosphere and the surrounding air temperature ____. on his tombstone, who was jimmie lee jackson trying to protect when he was killed? The total current density in a semiconductor is constant and equal to J10 A/cm2. The total current is composed of a hole drift current and an electron diffusion current. Assume that the hole concentration is constant and equal to 1016 cm3 and that the electron concentration is given by n(x) = 2x 1015c"1cm" where L = 15 m. The electron diffusion coefficient is D,-27 cm2/s and the hole mobility is ,-420 cm2/Vs. Calculate (a) the electron diffusion current density for x > 0, (b) the hole drift current density for >0, and (c) the required electric field for x >0. Give me a specific example of a time when you had to conform to a policy with which you did not agree Why are the kurds fighting so hard for the creation of kurdistan? 9687 rounded to the nearest thousand A student fires a 0.07 kg arrow at an object with mass m that is initially at rest on a frictionless surface. The speed of the arrow before the collision is 90 m/s. The speed when the arrow emerges from the object is v. What is the resulting velocity of the object? (4 pts)b) Is the collision between the arrow and the object elastic or inelastic? Include evidence to support your answer TRUE/FALSE. the recrystallization temperature is the temperature at which a metal recrystallizes, forming new grains instead of strain hardening. Carl wants to diagram a model of the flow of energy in a desert ecosystem. Will carl need to use a food chain or food web for the most accurate representation? Support your answer. what is called a leadership style that involves making managerial decisions without consulting others. [TRUE or FALSE] : governments can protect consumers from unsafe products by issuing a limit or a ban on such products. True/False?each different signal in a 1h nmr spectrum represents a different of hydrogen atom. hydrogen atoms that are give the same signal. Can you please help with this problem